Stay ahead of learning milestones! Enroll in a class over the summer!

G
Topic
First Poster
Last Poster
k a April Highlights and 2025 AoPS Online Class Information
jlacosta   0
Apr 2, 2025
Spring is in full swing and summer is right around the corner, what are your plans? At AoPS Online our schedule has new classes starting now through July, so be sure to keep your skills sharp and be prepared for the Fall school year! Check out the schedule of upcoming classes below.

WOOT early bird pricing is in effect, don’t miss out! If you took MathWOOT Level 2 last year, no worries, it is all new problems this year! Our Worldwide Online Olympiad Training program is for high school level competitors. AoPS designed these courses to help our top students get the deep focus they need to succeed in their specific competition goals. Check out the details at this link for all our WOOT programs in math, computer science, chemistry, and physics.

Looking for summer camps in math and language arts? Be sure to check out the video-based summer camps offered at the Virtual Campus that are 2- to 4-weeks in duration. There are middle and high school competition math camps as well as Math Beasts camps that review key topics coupled with fun explorations covering areas such as graph theory (Math Beasts Camp 6), cryptography (Math Beasts Camp 7-8), and topology (Math Beasts Camp 8-9)!

Be sure to mark your calendars for the following events:
[list][*]April 3rd (Webinar), 4pm PT/7:00pm ET, Learning with AoPS: Perspectives from a Parent, Math Camp Instructor, and University Professor
[*]April 8th (Math Jam), 4:30pm PT/7:30pm ET, 2025 MATHCOUNTS State Discussion
April 9th (Webinar), 4:00pm PT/7:00pm ET, Learn about Video-based Summer Camps at the Virtual Campus
[*]April 10th (Math Jam), 4:30pm PT/7:30pm ET, 2025 MathILy and MathILy-Er Math Jam: Multibackwards Numbers
[*]April 22nd (Webinar), 4:00pm PT/7:00pm ET, Competitive Programming at AoPS (USACO).[/list]
Our full course list for upcoming classes is below:
All classes run 7:30pm-8:45pm ET/4:30pm - 5:45pm PT unless otherwise noted.

Introductory: Grades 5-10

Prealgebra 1 Self-Paced

Prealgebra 1
Sunday, Apr 13 - Aug 10
Tuesday, May 13 - Aug 26
Thursday, May 29 - Sep 11
Sunday, Jun 15 - Oct 12
Monday, Jun 30 - Oct 20
Wednesday, Jul 16 - Oct 29

Prealgebra 2 Self-Paced

Prealgebra 2
Sunday, Apr 13 - Aug 10
Wednesday, May 7 - Aug 20
Monday, Jun 2 - Sep 22
Sunday, Jun 29 - Oct 26
Friday, Jul 25 - Nov 21

Introduction to Algebra A Self-Paced

Introduction to Algebra A
Monday, Apr 7 - Jul 28
Sunday, May 11 - Sep 14 (1:00 - 2:30 pm ET/10:00 - 11:30 am PT)
Wednesday, May 14 - Aug 27
Friday, May 30 - Sep 26
Monday, Jun 2 - Sep 22
Sunday, Jun 15 - Oct 12
Thursday, Jun 26 - Oct 9
Tuesday, Jul 15 - Oct 28

Introduction to Counting & Probability Self-Paced

Introduction to Counting & Probability
Wednesday, Apr 16 - Jul 2
Thursday, May 15 - Jul 31
Sunday, Jun 1 - Aug 24
Thursday, Jun 12 - Aug 28
Wednesday, Jul 9 - Sep 24
Sunday, Jul 27 - Oct 19

Introduction to Number Theory
Thursday, Apr 17 - Jul 3
Friday, May 9 - Aug 1
Wednesday, May 21 - Aug 6
Monday, Jun 9 - Aug 25
Sunday, Jun 15 - Sep 14
Tuesday, Jul 15 - Sep 30

Introduction to Algebra B Self-Paced

Introduction to Algebra B
Wednesday, Apr 16 - Jul 30
Tuesday, May 6 - Aug 19
Wednesday, Jun 4 - Sep 17
Sunday, Jun 22 - Oct 19
Friday, Jul 18 - Nov 14

Introduction to Geometry
Wednesday, Apr 23 - Oct 1
Sunday, May 11 - Nov 9
Tuesday, May 20 - Oct 28
Monday, Jun 16 - Dec 8
Friday, Jun 20 - Jan 9
Sunday, Jun 29 - Jan 11
Monday, Jul 14 - Jan 19

Intermediate: Grades 8-12

Intermediate Algebra
Monday, Apr 21 - Oct 13
Sunday, Jun 1 - Nov 23
Tuesday, Jun 10 - Nov 18
Wednesday, Jun 25 - Dec 10
Sunday, Jul 13 - Jan 18
Thursday, Jul 24 - Jan 22

Intermediate Counting & Probability
Wednesday, May 21 - Sep 17
Sunday, Jun 22 - Nov 2

Intermediate Number Theory
Friday, Apr 11 - Jun 27
Sunday, Jun 1 - Aug 24
Wednesday, Jun 18 - Sep 3

Precalculus
Wednesday, Apr 9 - Sep 3
Friday, May 16 - Oct 24
Sunday, Jun 1 - Nov 9
Monday, Jun 30 - Dec 8

Advanced: Grades 9-12

Olympiad Geometry
Tuesday, Jun 10 - Aug 26

Calculus
Tuesday, May 27 - Nov 11
Wednesday, Jun 25 - Dec 17

Group Theory
Thursday, Jun 12 - Sep 11

Contest Preparation: Grades 6-12

MATHCOUNTS/AMC 8 Basics
Wednesday, Apr 16 - Jul 2
Friday, May 23 - Aug 15
Monday, Jun 2 - Aug 18
Thursday, Jun 12 - Aug 28
Sunday, Jun 22 - Sep 21
Tues & Thurs, Jul 8 - Aug 14 (meets twice a week!)

MATHCOUNTS/AMC 8 Advanced
Friday, Apr 11 - Jun 27
Sunday, May 11 - Aug 10
Tuesday, May 27 - Aug 12
Wednesday, Jun 11 - Aug 27
Sunday, Jun 22 - Sep 21
Tues & Thurs, Jul 8 - Aug 14 (meets twice a week!)

AMC 10 Problem Series
Friday, May 9 - Aug 1
Sunday, Jun 1 - Aug 24
Thursday, Jun 12 - Aug 28
Tuesday, Jun 17 - Sep 2
Sunday, Jun 22 - Sep 21 (1:00 - 2:30 pm ET/10:00 - 11:30 am PT)
Monday, Jun 23 - Sep 15
Tues & Thurs, Jul 8 - Aug 14 (meets twice a week!)

AMC 10 Final Fives
Sunday, May 11 - Jun 8
Tuesday, May 27 - Jun 17
Monday, Jun 30 - Jul 21

AMC 12 Problem Series
Tuesday, May 27 - Aug 12
Thursday, Jun 12 - Aug 28
Sunday, Jun 22 - Sep 21
Wednesday, Aug 6 - Oct 22

AMC 12 Final Fives
Sunday, May 18 - Jun 15

F=ma Problem Series
Wednesday, Jun 11 - Aug 27

WOOT Programs
Visit the pages linked for full schedule details for each of these programs!


MathWOOT Level 1
MathWOOT Level 2
ChemWOOT
CodeWOOT
PhysicsWOOT

Programming

Introduction to Programming with Python
Thursday, May 22 - Aug 7
Sunday, Jun 15 - Sep 14 (1:00 - 2:30 pm ET/10:00 - 11:30 am PT)
Tuesday, Jun 17 - Sep 2
Monday, Jun 30 - Sep 22

Intermediate Programming with Python
Sunday, Jun 1 - Aug 24
Monday, Jun 30 - Sep 22

USACO Bronze Problem Series
Tuesday, May 13 - Jul 29
Sunday, Jun 22 - Sep 1

Physics

Introduction to Physics
Wednesday, May 21 - Aug 6
Sunday, Jun 15 - Sep 14
Monday, Jun 23 - Sep 15

Physics 1: Mechanics
Thursday, May 22 - Oct 30
Monday, Jun 23 - Dec 15

Relativity
Sat & Sun, Apr 26 - Apr 27 (4:00 - 7:00 pm ET/1:00 - 4:00pm PT)
Mon, Tue, Wed & Thurs, Jun 23 - Jun 26 (meets every day of the week!)
0 replies
jlacosta
Apr 2, 2025
0 replies
Putnam 2018 B4
62861   22
N 10 minutes ago by Ilikeminecraft
Given a real number $a$, we define a sequence by $x_0 = 1$, $x_1 = x_2 = a$, and $x_{n+1} = 2x_nx_{n-1} - x_{n-2}$ for $n \ge 2$. Prove that if $x_n = 0$ for some $n$, then the sequence is periodic.
22 replies
62861
Dec 2, 2018
Ilikeminecraft
10 minutes ago
Putnam 2006 B1
Kent Merryfield   54
N 13 minutes ago by Ilikeminecraft
Show that the curve $x^{3}+3xy+y^{3}=1$ contains only one set of three distinct points, $A,B,$ and $C,$ which are the vertices of an equilateral triangle.
54 replies
Kent Merryfield
Dec 4, 2006
Ilikeminecraft
13 minutes ago
Putnam 2015 B4
Kent Merryfield   23
N 29 minutes ago by Ilikeminecraft
Let $T$ be the set of all triples $(a,b,c)$ of positive integers for which there exist triangles with side lengths $a,b,c.$ Express \[\sum_{(a,b,c)\in T}\frac{2^a}{3^b5^c}\]as a rational number in lowest terms.
23 replies
Kent Merryfield
Dec 6, 2015
Ilikeminecraft
29 minutes ago
Subset Ordered Pairs of {1, 2, ..., 10}
ahaanomegas   10
N 42 minutes ago by Ilikeminecraft
Source: Putnam 1990 A6
If $X$ is a finite set, let $X$ denote the number of elements in $X$. Call an ordered pair $(S,T)$ of subsets of $ \{ 1, 2, \cdots, n \} $ $ \emph {admissible} $ if $ s > |T| $ for each $ s \in S $, and $ t > |S| $ for each $ t \in T $. How many admissible ordered pairs of subsets $ \{ 1, 2, \cdots, 10 \} $ are there? Prove your answer.
10 replies
ahaanomegas
Jul 12, 2013
Ilikeminecraft
42 minutes ago
No more topics!
Putnam 2019 A1
awesomemathlete   31
N Mar 2, 2025 by HamstPan38825
Source: 2019 William Lowell Putnam Competition
Determine all possible values of $A^3+B^3+C^3-3ABC$ where $A$, $B$, and $C$ are nonnegative integers.
31 replies
awesomemathlete
Dec 10, 2019
HamstPan38825
Mar 2, 2025
Putnam 2019 A1
G H J
G H BBookmark kLocked kLocked NReply
Source: 2019 William Lowell Putnam Competition
The post below has been deleted. Click to close.
This post has been deleted. Click here to see post.
awesomemathlete
120 posts
#1 • 2 Y
Y by FaThEr-SqUiRrEl, Adventure10
Determine all possible values of $A^3+B^3+C^3-3ABC$ where $A$, $B$, and $C$ are nonnegative integers.
This post has been edited 2 times. Last edited by awesomemathlete, Dec 22, 2019, 8:33 AM
Z K Y
The post below has been deleted. Click to close.
This post has been deleted. Click here to see post.
awesomemathlete
120 posts
#2 • 2 Y
Y by ashrith9sagar_1, Adventure10
Let $f(A,B,C)=A^3+B^3+C^3-3ABC$. Note that $f(n,n,n)=0$, $f(n+1,n,n)=3n+1$, $f(n-1,n,n)=3n-1$, and $f(n+1,n,n-1)=9n$. Thus $\boxed{\forall m\in\mathbb{Z}_{\ge 0}\text{s.t.}m\not\equiv 3,6\pmod{9}}$ there exist nonnegative integers $A$,$B$, and $C$ such that $A^3+B^3+C^3-3ABC=m$. Note that $A^3\equiv A\pmod{3}$ implies $f(A,B,C)\equiv A+B+C\pmod{3}$ but then if $f(A,B,C)=(A+B+C)(A^2+B^2+C^2-AB-BC-CA)$ is congruent to $0$ modulo $3$ it must be congruent to $0$ modulo $9$ as well as we inspect on the $4$ cases where the set of residues for $A$, $B$, and $C$ is $\{0,0,0\}$,$\{1,1,1\}$,$\{-1,-1,-1\}$, or $\{1,0,-1\}$.
$\square$
Z K Y
The post below has been deleted. Click to close.
This post has been deleted. Click here to see post.
The_Maitreyo1
1039 posts
#4 • 2 Y
Y by Adventure10, Mango247
Honestly speaking, this A1 seems to be a bit more difficult in comparison to last years A1. In general, how was the paper?
Z K Y
The post below has been deleted. Click to close.
This post has been deleted. Click here to see post.
scrabbler94
7553 posts
#5 • 2 Y
Y by Adventure10, Mango247
slightly alternate solution (sketch)
Z K Y
The post below has been deleted. Click to close.
This post has been deleted. Click here to see post.
klevasseur
1 post
#6 • 2 Y
Y by Adventure10, Mango247
One of my students told me he got this but failed to prove that the range was only the nonnegatives that are not congruent to 3 or 6 mod 9. How many points do you think he will earn?
Z K Y
The post below has been deleted. Click to close.
This post has been deleted. Click here to see post.
scrabbler94
7553 posts
#7 • 1 Y
Y by Adventure10
klevasseur wrote:
One of my students told me he got this but failed to prove that the range was only the nonnegatives that are not congruent to 3 or 6 mod 9. How many points do you think he will earn?

Most likely zero or one points (Putnam is very harsh on grading; scores other than 0, 1, 9, 10 are uncommon).
Z K Y
The post below has been deleted. Click to close.
This post has been deleted. Click here to see post.
tpatpatpa
1 post
#8 • 1 Y
Y by Adventure10
I got the correct answer and I proved everything except that the range is nonnegative (I didn't factor the expression). How many points do you think I'll get?
Z K Y
The post below has been deleted. Click to close.
This post has been deleted. Click here to see post.
a1267ab
223 posts
#9 • 8 Y
Y by trumpeter, scrabbler94, 62861, MarkBcc168, khina, IAmTheHazard, Adventure10, centslordm
https://artofproblemsolving.com/community/c6h1648142
https://artofproblemsolving.com/community/c6h1758384
https://artofproblemsolving.com/community/c6h1758561
https://artofproblemsolving.com/community/c4h1652099

Congratulations to CantonMathGuy for getting a problem onto the Putnam!
Z K Y
The post below has been deleted. Click to close.
This post has been deleted. Click here to see post.
trumpeter
3332 posts
#10 • 2 Y
Y by MarkBcc168, Adventure10
The answers are any integer $n$ with $\boxed{\nu_3(n)\neq1}$. Let $D(A,B,C)=A^3+B^3+C^3-3ABC$. These constructions suffice:
\begin{align*}
	D(0,0,0) &= 0 \\
	D(k+2,k+1,k) &= 9k+9 \\
	D(k+1,k,k) &= 3k+1 \\
	D(k+1,k+1,k) &= 3k+2
\end{align*}Clearly $D(A,B,C)$ is a nonnegative integer (AM-GM, $\mathbb{Z}$ is a ring). So it suffices to show that $D\equiv3,6\pmod9$ is impossible.

Assume that $3\mid D$. If $3\mid ABC$ then $D\equiv A^3+B^3\pmod9$, so $D\equiv0\pmod9$. So $3\nmid ABC$. Then $A^3+B^3+C^3$ is divisible by $3$, so $(A,B,C)\equiv\pm(1,1,1)\pmod3$. But then $D\equiv0\pmod9$.
Z K Y
The post below has been deleted. Click to close.
This post has been deleted. Click here to see post.
yayups
1614 posts
#11 • 4 Y
Y by Mahi07, guptaamitu1, Adventure10, Mango247
Let $T=A^3+B^3+C^3-3ABC$. We claim $T$ can take on any nonnegative value that isn't $3\pmod{9}$ or $6\pmod{9}$. Note that plugging in $(x,x,x+1)$ gives $3x+1$, plugging in $(x,x+1,x+1)$ gives $3x+2$, plugging in $(x,x+1,x+2)$ gives $9x+9$, and plugging in $(0,0,0)$ gives $0$.

It suffices then to show that if $T$ is divisible by $3$, then it is also divisible by $9$. This is true because we can factor $T$ as
\[T=(A+B+C)((A+B+C)^2-3(AB+BC+CA)).\]It is evident that if $3$ divides one of the factors, it must divide the other, so $3\mid T\implies 9\mid T$, as desired. This completes the solution.
Z K Y
The post below has been deleted. Click to close.
This post has been deleted. Click here to see post.
hoeij
35 posts
#12 • 2 Y
Y by Adventure10, Mango247
The Norm in number theory is:

If $f(x) \in \mathbb{Q}[x]$ and $r$ is an element of $R := \mathbb{Q}[x]/(f(x))$, then the Norm of $r$ is:
(1) the product of $r$ taken over all $x \in $ roots of $f(x)$
(2) the determinant of the $\mathbb{Q}$-linear map $R \rightarrow R$ given by multiplication by $r$.

Observation: $X := A^3+B^3+C^3-3ABC$ is the Norm of $r := A + Bx + Cx^2$ when we take $f(x) := x^3 - 1$. Since $f(x)$ has two factors over $\mathbb{Q}$ and three factors over $\mathbb{C}$, the same will also be true for $X$. The fact that $\mathbb{N}[x]/(x^3-1)$ is closed under multiplication implies that its set of Norms (the set of $X$ values in question A1) is also closed under multiplication. This may be a good way to construct similar exercises.
Z K Y
The post below has been deleted. Click to close.
This post has been deleted. Click here to see post.
Math-wiz
6107 posts
#13 • 1 Y
Y by Imayormaynotknowcalculus
This is the example 6 of Titu Andreescu's book on Diophantine equations :?
Z K Y
The post below has been deleted. Click to close.
This post has been deleted. Click here to see post.
Mahi07
2 posts
#14
Y by
@Math-wiz no thats a totally different sum which you are talking about. Thats far more easier than this one.
Z K Y
The post below has been deleted. Click to close.
This post has been deleted. Click here to see post.
GeronimoStilton
1521 posts
#15
Y by
Thanks Titu!

Write
\[a^3+b^3+c^3-3abc=\frac 12\cdot (a+b+c)\sum_{\mbox{cyc}}(a-b)^2.\]So by taking $b=a,c=a+1$ we can achieve any $3a+1$ and by taking $b=c=a+1$ we can achieve any $3a+2$. It remains to figure out which multiples of $3$ we can achieve. We can achieve multiples of $9$ by setting $b=a+1,c=a+2$ to get $3(3a+3)$.

Now it remains to see when we can get non-multiples of $9$. Then exactly one of $(a+b+c)$ and $\displaystyle\sum_{\mbox{cyc}}(a-b)^2$ is a multiple of $3$. If the latter is the case: if some two of $a,b,c$ are the same mod $3$, then all are, but this cannot occur so $a,b,c$ are all distinct modulo $3$. This is also a contradiction. So then the first case holds. Then $a,b,c$ are all congruent modulo $3$ or all distinct modulo $3$ which again ends up causing a contradiction.

To summarize, non multiples of $3$ and multiples of $9$ are achievable.
Z K Y
The post below has been deleted. Click to close.
This post has been deleted. Click here to see post.
OlympusHero
17020 posts
#16
Y by
Is it possible to solve this via the factorization $(A+B+C)(A^2+B^2+C^2-AB-AC-BC)=A^3+B^3+C^3-3ABC$ and Introduction to Number Theory level Modular Arithmetic? If so, can someone provide a basic solution like that? Just wondering.
Z K Y
The post below has been deleted. Click to close.
This post has been deleted. Click here to see post.
Archeon
5970 posts
#17 • 1 Y
Y by Emo916math
Rewrite as $(A+B+C)((A+B+C)^2-3(AB-AC-BC))$. Then the first term is divisible by $3$ iff the second is. This means that $3,6$ mod $9$ are impossible, and then you show the rest work.
Z K Y
The post below has been deleted. Click to close.
This post has been deleted. Click here to see post.
OlympusHero
17020 posts
#18 • 1 Y
Y by Mango247
And to show the rest work you just construct $A,B,C$ as residues $\pmod 9$ such that everything else works?
Z K Y
The post below has been deleted. Click to close.
This post has been deleted. Click here to see post.
Archeon
5970 posts
#19
Y by
More or less, yeah. A construction like the one in post #10 suffices.
Z K Y
The post below has been deleted. Click to close.
This post has been deleted. Click here to see post.
OlympusHero
17020 posts
#20 • 1 Y
Y by Emo916math
Thanks to Archeon and trumpeter for providing assistance with this solution.

Solution
Z K Y
The post below has been deleted. Click to close.
This post has been deleted. Click here to see post.
jasperE3
11248 posts
#21
Y by
Oops, separate constructions for all$\pmod9$ residues.
Suppose that $A^3+B^3+C^3-3ABC$ is a multiple of $3$. We claim that $3\mid A+B+C$ and $3\mid A^2+B^2+C^2-AB-BC-CA$, which together imply that $9\mid A^3+B^3+C^3-3ABC$.

Then:
$$0\equiv A^3+B^3+C^3-3ABC\equiv A^3+B^3+C^3\equiv A+B+C\pmod3$$by FLT, so $3\mid A+B+C$.

We check that if $(A\pmod3,B\pmod3,C\pmod3)\in\{(0,0,0),(1,1,1),(2,1,0)\}$ and permutations then $3\mid A^2+B^2+C^2-AB-BC-CA$. These are the only possibilities since $3\mid A+B+C$.

So $A^3+B^3+C^3-3ABC$ can only be $0,1,2,4,5,7,8\pmod9$. We can show that all of these are achievable:
By setting $(A,B,C)=(1,1,1)$, $0$ is possible.
By setting $(A,B,C)=(n+2,n+1,n)$, $0\pmod9$ (except for $0$) is possible.
By setting $(A,B,C)=(3n+1,3n,3n)$, $1\pmod9$ is possible.
By setting $(A,B,C)=(3n+1,3n+1,3n)$, $2\pmod9$ is possible.
By setting $(A,B,C)=(3n+4,3n+3,3n+3)$, $4\pmod9$ is possible.
By setting $(A,B,C)=(3n+4,3n+4,3n+3)$, $5\pmod9$ is possible.
By setting $(A,B,C)=(3n+7,3n+6,3n+6)$, $7\pmod9$ is possible.
By setting $(A,B,C)=(3n+7,3n+7,3n+6)$, $8\pmod9$ is possible.
Z K Y
The post below has been deleted. Click to close.
This post has been deleted. Click here to see post.
megarnie
5596 posts
#22
Y by
The answer is $\boxed{\text{all }n\in \mathbb{Z}_{\ge 0}, n\not\equiv \{3,6\}\pmod 9}$.

Another way to write the solution set is $\boxed{\text{all nonnegative integers }n \text{ such that }\nu_3(n)\ne 1}$.

Let $f(A,B,C)=A^3+B^3+C^3-3ABC$.

We will prove this by considering cases on $f(A,B,C)$ in $\pmod 9$, and then show $f(A,B,C)\ge 0$.

Case 1: $f(A,B,C)=3k+1$ (indeed, this covers $1,4,7$ mod $9$).
Clearly $(k+1,k,k)$ works.

Case 2: $f(A,B,C)=3k+2$ (indeed, this covers $2,5,8$ mod $9$).
Clearly $(A,B,C)=(k+1,k+1,k)$ works.

Case 3: $f(A,B,C)=9k$
For $k\ge 1$, set $(A,B,C)=(k-1,k,k+1)$. This works. For $k=0$, set $(A,B,C)=(0,0,0)$.

Case 4: $f(A,B,C)=9k+3$ or $9k+6$.
We have \[f(A,B,C)=(A+B+C)(A^2+B^2+C^2-AB-BC-AC)=(A+B+C)((A+B+C)^2-3(AB+BC+AC))\]
Note that $3$ must divide $f(A,B,C)$ and $9\nmid f(A,B,C)$. If $3$ divides $A+B+C$, then $9$ divides $f(A,B,C)$, a contradiction. If $3\nmid f(A,B,C)$, then $3\nmid f(A,B,C)$, a contradiction.

So there are $f(A,B,C)\not\equiv \{3,6\}\pmod 9$.

Lemma (which finishes): $f(A,B,C)\ge 0$
We have \[f(A,B,C)=(A+B+C)(A^2+B^2+C^2-AB-BC-AC)\]
Note the following fact that over nonnegative reals, $a^2+b^2\ge 2ab$, because $a^2+b^2-2ab=(a-b)^2\ge 0$.

Now $2(A^2+B^2+C^2)=(A^2+B^2)+(B^2+C^2)+(A^2+C^2)\ge 2(AB+BC+AC)$.

Thus, $A^2+B^2+C^2\ge AB+BC+AC$, which implies $f(A,B,C)$ is nonnegative. $\blacksquare$
Z K Y
The post below has been deleted. Click to close.
This post has been deleted. Click here to see post.
Sagnik123Biswas
420 posts
#23
Y by
All numbers $n$ that are either $1$ or $2$ mod 3 or $0$ mod 9.

If $n = 3k+1$, then $A = k, B = k, C = k+1$ is a construction
If $n = 3k-1$, then $A = k, B = k, C = k-1$ is a construction
If $n = 9k$, then $A = k-1, B = k, C = k+1$ is a construction

If $n$ is divisible by $3$, it must be divisible by $9$. This can be seen by factoring $A^3+B^3+C^3-3ABC$ and looking over all residue combinations of $A, B, C$ módulo $3$ that allow the expression to be divisible by $3$. Thus, these are all the cases.
Z K Y
The post below has been deleted. Click to close.
This post has been deleted. Click here to see post.
YaoAOPS
1530 posts
#24
Y by
Note that this expression is strictly positive by AM-GM.

Claim: We can construct all $n \ge 0$ with $\nu_3(n) \ne 1$.
Proof. By taking $a = b = c$, we get $0$.
By taking $a = t, b = t, c = t + 1$, we get $3t + 1$. Similarly, by taking $a = t, b = t + 1, c = t + 1$, we get $3t + 2$.
By taking $a = (t-1), b = t, c = (t+1)$, we get $9t$. $\blacksquare$

Claim: We can't construct $n = 3k$ and $\gcd(k, 3) = 1$.
Proof. Take $\pmod{9}$ to get the result. $\blacksquare$
Z K Y
The post below has been deleted. Click to close.
This post has been deleted. Click here to see post.
Pyramix
419 posts
#25
Y by
Let $f(a,b,c)=a^3+b^3+c^3-3abc$.
Plug $f(n+1,n,n)=3n+1$ while $f(n-1,n,n)=3n-1$. Finally, $f(n+1,n,n-1)=9n$. We now show that it is impossible for $f(a,b,c)\equiv 3,6\pmod{9}$.
Note that $3\nmid\gcd(a,b,c)$ and $3\mid f(a,b,c)$ is possible only if:
  • $a\equiv b\equiv c\pmod{3}$,
  • $3\mid a,b+c$.
If $a\equiv b\equiv c\equiv 1\pmod{3}$, then $a^3\equiv b^3\equiv c^3\equiv 1\pmod{9}$ and $abc\equiv1\pmod{3}$ which means $3abc\equiv3\pmod{9}$. Hence, $9\mid f(a,b,c)$.
Similarly one can check for $a\equiv b\equiv c\equiv -1\pmod{3}$.
Now, if $3\mid a$ then $9\mid abc$. If $3\mid b+c$ then $9\mid b^3+c^3=(b+c)^3-3bc(b+c)$, while $9\mid a$. Hence, $9\mid f(a,b,c)$.

So, if $3\mid f(a,b,c)$ then $9\mid f(a,b,c)$, as claimed.

So, only $\{9n-9,3n-1,3n+1\}$ work for every $n\in\mathbb{N}$.
Z K Y
The post below has been deleted. Click to close.
This post has been deleted. Click here to see post.
sanyalarnab
930 posts
#26
Y by
I claim that all $n \in \{0,1,2,4,5,7,8\} \pmod 9$ work. Let's see the constructs for these values:
$(A,B,C) \equiv (n,n,n+1) \implies A^3+B^3+C^3-3ABC=3n+1$(constitutes for $1,4,7 \pmod 9$) for all non negative integers $n$.
$(A,B,C) \equiv (n,n,n-1) \implies A^3+B^3+C^3-3ABC=3n-1$(constitutes for $2,5,8 \pmod 9$) for all positive integers $n$.
$(A,B,C) \equiv (n,n+2,n+1) \implies A^3+B^3+C^3-3ABC=9n+9$(constitutes for $0 \pmod 9$) for all non negative integers $n$.
$(A,B,C) \equiv (0,0,0) \implies A^3+B^3+C^3-3ABC=0$
Now we note that $$3|A+B+C \leftrightarrow 3|(A+B+C)^2-3(AB+BC+CA)$$Thus $v_3(A^3+B^3+C^3-3ABC) \neq 1 \implies 3,6 \pmod 9$ are not achievable. $\blacksquare$
This post has been edited 1 time. Last edited by sanyalarnab, Apr 13, 2024, 12:13 PM
Z K Y
The post below has been deleted. Click to close.
This post has been deleted. Click here to see post.
lifeismathematics
1188 posts
#27
Y by
cool problem!

We claim that all numbers other than $3 , 6$ $\pmod 9$ works.

$\emph{Proof:-}$ First we prove if $3|A^3+B^3+C^3-3ABC$ then $9|A^3+B^3+C^3-3ABC$ , notice $A^3+B^3+C^3-3ABC=(A+B+C)(A^2+B^2+C^2-AB-BC-CA)$ , now $3|(A+B+C)\left((A+B+C)^2-3(AB+BC+CA)\right)$ , now that forces $3|A+B+C$ , now that implies $A^3+B^3+C^3-3ABC=9k_{1}k_{2}$ for some $k_{1} , k_{2} \in \mathbb{Z}$ and hence $9|A^3+^3+C^3-3ABC$ , so clearly $3,6$ $\pmod9 $ are not possible. $\square$

Now we give constructions for other numbers , we notice $(A,B,C)=(n,n+2,n+5)$ gives $A^3+B^3+C^3-3ABC=3n+7$ under $\pmod 9$ , which gives $1,4,7$ as residues under $\pmod 9$.

for $(A,B,C)=(n,n,n-1)$ we get $A^3+B^3+C^3-3ABC=3n-1$ which gives $2,5,8$ residues under $\pmod 9$ , and finally for $(3n, 6n,9n)$ we get $0$ $\pmod 9$ , which finally accounts all possible values of $A^3+B^3+C^3-3ABC$ for non negative integers $A,B,C$. $\square$
Z K Y
The post below has been deleted. Click to close.
This post has been deleted. Click here to see post.
popop614
271 posts
#28
Y by
???????????????????

All integers $n$ with $\nu_3(n) \neq 1$ work. To construct you just take $(a, a, a+1)$, $(a+1, a+1, a)$, and $(a, a+1, a+2)$.

Notice that $3 \mid a + b + c \iff 3 \mid a^2 + b^2 + c^2 + 2(ab + bc + ca) \iff 3 \mid a^2 + b^2 + c^2 - ab - bc - ca$. Therefore if $3$ divides the expression then $9$ does. We are done.
Z K Y
The post below has been deleted. Click to close.
This post has been deleted. Click here to see post.
Hello_Kitty
1889 posts
#29
Y by
PUTNAM 2019
Attachments:
Z K Y
The post below has been deleted. Click to close.
This post has been deleted. Click here to see post.
Bluesoul
894 posts
#30
Y by
$a^3+b^3+c^3-3abc=(a+b+c)(a^2+b^2+c^2-ab-ac-bc)=(a+b+c)((a+b+c)^2-3(ab+bc+ac))=(a+b+c)^3-3(a+b+c)(ab+bc+ac)=S$

If $a+b+c$ is a multiple of 3, denote $a+b+c=3k, 27k^3-9k(ab+bc+ac)=S$ must be a multiple of $9$.

If $a+b+c\equiv 1\pmod{3}$, denote $a+b+c=3k+1, (3k+1)^3-3(3k+1)(ab+bc+ac)\equiv 1-3(ab+bc+ac)\pmod{9}$

We have three cases, $(1,0,0), (2,2,0), (2,1,1)$, where the integers in the parenthesis are the remainders of $a,b,c$ divided by $3$. Test three cases, $S\equiv 1,4,7\pmod 9$

Similarly, when $a+b+c\equiv 2\pmod{3}, a+b+c=3k+2$, $S\equiv 8-6(ab+bc+ac)\pmod{9}$

Then three cases are $(1,1,0), (2,2,1), (2,0,0)$ yielding $S\equiv 2,5,8 \pmod{9}$

Thus, $S\equiv 0,1,2,4,5,7,8 \pmod{9}$ as desired.
Z K Y
The post below has been deleted. Click to close.
This post has been deleted. Click here to see post.
megahertz13
3183 posts
#31
Y by
The answer is $$\boxed{n\not\equiv 3,6\pmod 9}.$$
First, one can check that using:
$(a,b,c)=(k+1,k,k)$ gives $n=3k+1$,
$(a,b,c)=(k+1,k+1,k)$ gives $n=3k+2$, and
$(a,b,c)=(k+2,k+1,k)$ gives $n=9k+9$. Letting $k$ vary gives all solutions.

Since cubes are in $\{-1,0,1\}$, $$n\not\equiv 3,6\pmod 9$$follows by inspection.
Z K Y
The post below has been deleted. Click to close.
This post has been deleted. Click here to see post.
emi3.141592
71 posts
#32
Y by
Observe that
$$
(n+1)^3 + n^3 + n^3 - 3(n+1)n^2 = 3n + 1,
$$$$
(n-1)^3 + n^3 + n^3 - 3(n-1)n^2 = 3n - 1,
$$$$
(n-1)^3 + n^3 + (n+1)^3 - 3(n-1)n(n+1) = 9n.
$$Thus, we can obtain all integers $m$ such that $m \not\equiv 3,6 \pmod{9}$.

To conclude, observe that
$$
3 \mid A^3 + B^3 + C^3 - 3ABC \overset{\text{FLT}}{\implies} 3 \mid A + B + C,
$$$$
\implies 3 \mid (A+B+C)^2 \implies 3 \mid A^2 + B^2 + C^2 - AB - BC - CA,
$$$$
\implies 9 \mid (A+B+C)(A^2 + B^2 + C^2 - AB - BC - CA) = A^3 + B^3 + C^3 - 3ABC.
$$Thus, we cannot obtain numbers that are $3$ or $6 \pmod{9}$, and we are done.
This post has been edited 1 time. Last edited by emi3.141592, Jan 29, 2025, 7:55 PM
Z K Y
The post below has been deleted. Click to close.
This post has been deleted. Click here to see post.
HamstPan38825
8857 posts
#33 • 1 Y
Y by teomihai
All integers that are not $3$ or $6$ mod $9$. Indeed, by setting $(a, b, c) = (a-1, a, a+1)$ we get $9a$ which covers all multiples of $9$, and by setting $(a, b, c) = (a, a, a \pm 1)$ we get $3a \pm 1$ which covers all non-multiples of $3$.

So it suffices to show that $3$ or $6$ mod $9$ integers fail. This just stems from the observation that
\[a^2+b^2+c^2-ab-bc-ca = (a+b+c)^2 - 3(ab+bc+ca) \equiv (a+b+c)^2 \pmod 3\]so either $a^3+b^3+c^3 -3abc = (a+b+c)(a^2+b^2+c^2-ab-bc-ca)$ is the product of two multiples of $3$ or two numbers relatively prime to $3$.
Z K Y
N Quick Reply
G
H
=
a